What is the region of convergence for the Laurent series?

Click For Summary
The discussion focuses on determining the region of convergence for a Laurent series composed of two parts. The first series, with negative exponents, converges for large values of |z| and does not converge at z=0, establishing the inner radius. The second series converges everywhere, providing the outer boundary of convergence. By substituting z with x-1, the first series is transformed, leading to a radius of convergence of |z| > 2. Ultimately, the overall region of convergence for the entire series is established as |z| > 2.
mattmns
Messages
1,121
Reaction score
5
This is more of a general question than a specific question.
----------
Find the annulus of convergence for the Laurent series
\sum_{n=-\infty}^{-1} \left( \frac{z}{2} \right)^n + \sum_{n=0}^\infty \frac{z^n}{n!}
--------

I know what to do for the second series, but I am not sure about the first series. The main thing that is bugging me is the -\infty to -1. In general, how do I deal with such a series? Is there something special that needs to be done, or do I treat it the same as a series from 0 to \infty? Thanks!
 
Physics news on Phys.org
An "annulus" is the region between two concentric circles. You are really looking for two "radii of convergence". It should be clear to you that if |z| is large, there will be no problem with thefirst series (where z has negative exponent) but might with the second. The series with the positive exponents will give you the outer boundary of convergence but converges everywhere inside it. (Actually, you have probably noticed that the second series is just the standard series for ez and converges for all z.)
It is the first series, with negative exponents, that will not converge for z= 0 but will converge for |z| large that will give you the "inner" radius.

Do this: replace z with x-1 so that
\sum_{n=-\infty}^{-1} \left( \frac{z}{2} \right)^n
becomes
\sum_{n= 1}^{\infty} \left(2x \right)^n

What is the radius of convergence of that? Now convert back to z= x-1.
 
Awesome idea, thanks!

The region of convergence for the new series is |x| < 1/2 => |z| > 2

Hence the region of convergence for the whole series is |z| > 2 since the other series converges everywhere.
 
Question: A clock's minute hand has length 4 and its hour hand has length 3. What is the distance between the tips at the moment when it is increasing most rapidly?(Putnam Exam Question) Answer: Making assumption that both the hands moves at constant angular velocities, the answer is ## \sqrt{7} .## But don't you think this assumption is somewhat doubtful and wrong?

Similar threads

  • · Replies 3 ·
Replies
3
Views
1K
  • · Replies 5 ·
Replies
5
Views
2K
  • · Replies 3 ·
Replies
3
Views
1K
  • · Replies 3 ·
Replies
3
Views
2K
  • · Replies 2 ·
Replies
2
Views
2K
  • · Replies 2 ·
Replies
2
Views
1K
  • · Replies 7 ·
Replies
7
Views
2K
  • · Replies 2 ·
Replies
2
Views
2K
  • · Replies 1 ·
Replies
1
Views
2K
  • · Replies 6 ·
Replies
6
Views
2K